424 lines
17 KiB
TeX
424 lines
17 KiB
TeX
\documentclass[a4paper,12pt, table]{/media/documents/Cours/Prof/Enseignements/2014-2015/tools/style/classExamen}
|
|
\usepackage{/media/documents/Cours/Prof/Enseignements/2014-2015/2014_2015}
|
|
\usepackage{csvsimple}
|
|
|
|
% Title Page
|
|
\titre{}
|
|
% \seconde \premiereS \PSTMG \TSTMG
|
|
\classe{Seconde}
|
|
\date{11 février 2015}
|
|
\typedoc{Devoir Commun}
|
|
\duree{3h}
|
|
\ptpres{4}
|
|
|
|
\printanswers
|
|
|
|
|
|
\begin{document}
|
|
\titlepage
|
|
|
|
\begin{questions}
|
|
|
|
\question[10]
|
|
% Depuis repère 67 p 157
|
|
Un magasin a annoncé sa journée de promotion par une distribution de tracts sur lesquels était indiqué:
|
|
\begin{center}
|
|
\textit{Grande journée de promotion! Dépensez moins!}
|
|
\end{center}
|
|
|
|
\textbf{Partie 1}\\
|
|
Le tableau ci-dessous donne les montants en euros, arrondis à l'unité, des achats effectués par les 80 clients du magasin pendant une journée ordinaire.
|
|
|
|
\definecolor{lightgray}{gray}{0.9}
|
|
\rowcolors{1}{lightgray}{}
|
|
\begin{center}
|
|
\begin{tabular}{|*{8}{c|}}
|
|
\hline
|
|
2 &3 &5 &5 &5 &8 &8 &8\\
|
|
\hline
|
|
8 &10 &10 &10 &10 &10 &10 &10\\
|
|
\hline
|
|
11 &13 &14 &14 &14 &20 &20 &20\\
|
|
\hline
|
|
20 &20 &20 &21 &24 &24 &25 &26\\
|
|
\hline
|
|
30 &30 &30 &30 &30 &30 &31 &33\\
|
|
\hline
|
|
33 &35 &36 &38 &38 &38 &38 &38\\
|
|
\hline
|
|
39 &39 &39 &39 &39 &40 &40 &40\\
|
|
\hline
|
|
40 &40 &40 &40 &40 &40 &42 &42\\
|
|
\hline
|
|
42 &43 &43 &43 &44 &44 &45 &45\\
|
|
\hline
|
|
45 &45 &45 &46 &46 &47 &55 &60\\
|
|
\hline
|
|
\end{tabular}
|
|
\end{center}
|
|
|
|
\begin{parts}
|
|
%1pt
|
|
\part Quelle est la population concernée par cette étude statistique? Quel est le caractère étudié?
|
|
\begin{solution}
|
|
Dans cette étude statistique, la population est l'ensemble des clients et le caractère étudié est leurs depenses.
|
|
\end{solution}
|
|
%1pts
|
|
\part Tracer le tableau des effectifs de cette série statistique.
|
|
\begin{solution}
|
|
Tableau des effectifs
|
|
|
|
\TODO{à faire..}
|
|
\end{solution}
|
|
% J'aimerai qu'ils aient à tracer un histogramme ici!
|
|
\part
|
|
\begin{subparts}
|
|
%1pt
|
|
\subpart Déterminer le pourcentage de clients ayant effectué des achats pour un montant ne dépassant pas les 27\euro.
|
|
\begin{solution}
|
|
Pourcentage des clients ayant fait des achats pour moins de 27\euro
|
|
\begin{eqnarray*}
|
|
\frac{32}{80} & = & 0,4 = 40\%
|
|
\end{eqnarray*}
|
|
|
|
\end{solution}
|
|
%1pt
|
|
\subpart Déterminer le pourcentage de clients ayant effectué des achats entre 30\euro\; et 40\euro\; inclus.
|
|
\begin{solution}
|
|
Pourcentage des clients ayant fait des achats entre 30 et 40\euro.
|
|
\begin{eqnarray*}
|
|
\frac{30}{80} & = & 0,375 = 37,5\%
|
|
\end{eqnarray*}
|
|
\end{solution}
|
|
\end{subparts}
|
|
% 2pt
|
|
\part Calculer la moyenne de cette série statistique.
|
|
\begin{solution}
|
|
\TODO{à faire..}
|
|
\end{solution}
|
|
\part
|
|
\begin{subparts}
|
|
%1pt
|
|
\subpart Déterminer le minimum et le maximum de cette série statistique.
|
|
\begin{solution}
|
|
En lisant le tableau de valeurs:
|
|
\begin{itemize}
|
|
\item Mininum: 2
|
|
\item Maximum: 60
|
|
\end{itemize}
|
|
\end{solution}
|
|
%2pts
|
|
\subpart Déterminer la médiane de cette série statistique.
|
|
\begin{solution}
|
|
Médiane de cette série. Dans le sujet les données sont déjà rangées par ordre croissant.
|
|
|
|
Effectif total: 80
|
|
|
|
Position de la médiane: $\frac{80}{2} = 40$ Donc la médiane se trouver entre la 40 et la 41ième valeur. Donc $Me =33 $.
|
|
\end{solution}
|
|
%2pts
|
|
\subpart Déterminer les quartiles de cette série statistique.
|
|
\begin{solution}
|
|
Position du premier quartile: $\frac{1}{4} \times 80 = 20$. Donc le premier quartile se trouve entre la 20ième et la 21ième valeur. Donc $Q_1 = 14$.
|
|
|
|
Position du troisième quartile: $\frac{3}{4} \times 80 = 60$. Donc le troisième quartile se trouve entre la 60ième et la 61ième valeur. Donc $Q_3 = 40$.
|
|
\end{solution}
|
|
\end{subparts}
|
|
|
|
\begin{EnvFullwidth}
|
|
|
|
\textbf{Partie 2}\\
|
|
Un étude similaire a été faite sur 80 clients lors d'une journée de promotion. Cette étude a donné les résultats suivants:
|
|
\begin{itemize}
|
|
\item Moyenne: 50
|
|
\item Minimum: 5
|
|
\item Premier quartile: 45
|
|
\item Médiane: 55
|
|
\item Troisième quartile: 63
|
|
\item Maximum: 75
|
|
\end{itemize}
|
|
\end{EnvFullwidth}
|
|
%2pts
|
|
\part En utilisant les résultats des deux études statistiques, commentez le message publicitaire de ce magasin.
|
|
\begin{solution}
|
|
On remarque que mis à part le minimum, tous les indicateurs ($Q_1$, $Me$, $Q_3$ et max) sont supérieurs lors d'une journée de promotion que lors d'une journée ordinaire. Les clients dépensent donc plus un jour de promotion qu'un jour ordinaire. Le slogan n'est donc pas véridique.
|
|
\end{solution}
|
|
\end{parts}
|
|
|
|
%\pagebreak
|
|
|
|
\question[9]
|
|
Dans un plan muni d'un repère orthonormé $(O;I;J)$, on a placé les points
|
|
\begin{eqnarray*}
|
|
T(-2;-2) \qquad R(0;2) \qquad I(2;1)
|
|
\end{eqnarray*}
|
|
\begin{parts}
|
|
%1pt
|
|
\part Faire la figure et la remplir au fil des questions suivantes.
|
|
\begin{solution}
|
|
\begin{center}
|
|
\begin{tikzpicture}[scale = 0.7]
|
|
\draw (-5,-5) grid (5,5);
|
|
\repere{-5}{5}{-5}{5}
|
|
\draw (-2,-2) node {$\bullet$} node [above right] {$T$};
|
|
\draw (0,2) node {$\bullet$} node [above right] {$R$};
|
|
\draw (2,1) node {$\bullet$} node [above right] {$I$};
|
|
\draw (-2,-2) -- (0,2) -- (2,1) -- (-2,-2);
|
|
|
|
\draw (0,-0.5) node {$\bullet$} node [below left] {$A$};
|
|
\draw (-1,0) node {$\bullet$} node [above left] {$0$};
|
|
\draw (-1,0) circle (2.23);
|
|
\end{tikzpicture}
|
|
\end{center}
|
|
\end{solution}
|
|
%2pts
|
|
\part Calculer les longueurs des trois côtés du triangle $TRI$.
|
|
\begin{solution}
|
|
Longueur du segment $[TR]$
|
|
\begin{eqnarray*}
|
|
TR & = & \sqrt{(x_T - x_R)^2 + (y_T - y_R)^2}\\
|
|
TR &=& \sqrt{(-2 - 0)^2 + (-2 - 2)^2}\\
|
|
TR &=& \sqrt{4 + 16}\\
|
|
TR &=& \sqrt{20}
|
|
\end{eqnarray*}
|
|
Longueur du segment $[TI]$
|
|
\begin{eqnarray*}
|
|
TI & = & \sqrt{(x_T - x_I)^2 + (y_T - y_I)^2}\\
|
|
TI &=& \sqrt{(-2 - 2)^2 + (-2 - 1)^2}\\
|
|
TI &=& \sqrt{16 + 9}\\
|
|
TI &=& \sqrt{25} = 5
|
|
\end{eqnarray*}
|
|
Longueur du segment $[RI]$
|
|
\begin{eqnarray*}
|
|
RI & = & \sqrt{(x_R - x_I)^2 + (y_R - y_I)^2}\\
|
|
RI &=& \sqrt{(0 - 2)^2 + (2 - 1)^2}\\
|
|
RI &=& \sqrt{4 + 1}\\
|
|
RI &=& \sqrt{5} \approx 2,23
|
|
\end{eqnarray*}
|
|
|
|
\end{solution}
|
|
%2pts
|
|
\part Démontrer que le triangle $TRI$ est un triangle rectangle. Est-il isocèle?
|
|
\begin{solution}
|
|
Pour savoir si le triangle est rectangle, on veut vérifier sur $TR^2 + RI^2$ est égal à $TI^2$.
|
|
\begin{eqnarray*}
|
|
TI^2 & = & 5^2 = 25 \\
|
|
TR^2 + RI^2 & = & \sqrt{20}^2 + \sqrt{5}^2 = 20 + 5 = 25
|
|
\end{eqnarray*}
|
|
Donc on a bien $TI^2 = TR^2 + RI^2$, d'après le théorème de Pythagore, le triangle $TRI$ est rectangle en $R$.
|
|
|
|
Ce triangle n'est pas isocèle, car d'après la question précédente, il y a aucun coté qui fait la même longueur qu'un autre.
|
|
\end{solution}
|
|
%2pts
|
|
\part Calculer les coordonnées du point $A$ milieu du segment $[TI]$. Placer ce point sur le dessin.
|
|
\begin{solution}
|
|
Calcul des coordonnées du point $A$ milieu de $[TI]$.
|
|
\begin{eqnarray*}
|
|
x_A & = & \frac{x_T + x_I}{2} = \frac{-2 + 2}{2} = 0 \\
|
|
y_A & = & \frac{y_T + y_I}{2} = \frac{-2 + 1}{2} = \frac{-1}{2} \\
|
|
\end{eqnarray*}
|
|
Donc $A (0;\frac{1}{2})$.
|
|
|
|
\end{solution}
|
|
\part
|
|
\begin{subparts}
|
|
% 0,5pt
|
|
\subpart Tracer le cercle $\mathcal{C}$ de diamètre $[TR]$.
|
|
\begin{solution}
|
|
Voir graphique
|
|
\end{solution}
|
|
% 1pt
|
|
\subpart Calculer les coordonnées de son centre.
|
|
\begin{solution}
|
|
Le centre du cercle, on l'appelle $O$, est le milieu de $[TR]$. On calcule ses coordonnées
|
|
\begin{eqnarray*}
|
|
x_0 & = & \frac{x_T + x_R}{2} = \frac{-2 + 0}{2} = -1 \\
|
|
y_0 & = & \frac{y_T + y_R}{2} = \frac{-2 + 2}{2} = 0
|
|
\end{eqnarray*}
|
|
\end{solution}
|
|
% 0,5
|
|
\subpart Calculer la mesure $r$ de son rayon.
|
|
\begin{solution}
|
|
Le rayon d'un cercle est égal à la moitier du diamètre $TR$. Donc
|
|
\begin{eqnarray*}
|
|
r & = & \frac{TR}{2} = \frac{\sqrt{20}}{2} \approx 2,23
|
|
\end{eqnarray*}
|
|
|
|
\end{solution}
|
|
\end{subparts}
|
|
\end{parts}
|
|
|
|
\pagebreak
|
|
\question[5]
|
|
Placer les points suivants sur le plan ci-dessous en laissant les traits de construction.
|
|
\begin{center}
|
|
\begin{tikzpicture}[scale=1]
|
|
\coordinate (A) at (4,5);
|
|
\coordinate (B) at (5,7);
|
|
\coordinate (C) at (6,4);
|
|
\coordinate (D) at (3,7);
|
|
\draw (0,0) grid (10,10);
|
|
\draw (A) node {$\bullet$} node[below left] {$A$};
|
|
\draw (B) node {$\bullet$} node[below right] {$B$};
|
|
\draw (C) node {$\bullet$} node[above right] {$C$};
|
|
\draw (D) node {$\bullet$} node[below left] {$D$};
|
|
\draw[very thick, ->] (2,7) -- (2,5) node[midway, above left] {$\vec{u}$};
|
|
|
|
\ifprintanswers
|
|
\draw[color = blue, very thick, ->] (B) -- (C) node[font=\tiny, sloped, midway, above] {$\vec{BC}$};
|
|
\draw[color = blue, very thick, ->] (A) --+ (1,-3) node[font=\tiny,sloped, midway, above] {$\vec{AU} = \vec{BC}$};
|
|
\draw[color = blue] (5,2) node {$\bullet$} node[below left] {$U$};
|
|
|
|
\draw[color = red, very thick, ->] (C) -- (D) node[font=\tiny,sloped, midway, above] {$\vec{CD}$};
|
|
\draw[color = red, very thick, <-] (A) --+ (3,-3) node[font=\tiny,sloped, midway, above] {$\vec{VA} = \vec{CD}$};
|
|
\draw[color = red] (7,2) node {$\bullet$} node[above right] {$V$};
|
|
|
|
|
|
\draw[color = green, very thick, ->] (A) -- (C) node[font=\tiny,sloped, midway, above] {$\vec{AC}$};
|
|
\draw[color = green, very thick, ->] (B) --+ (-2,1) node[font=\tiny,sloped, midway, above] {$\vec{BW} = -\vec{AC}$};
|
|
\draw[color = green] (3,8) node {$\bullet$} node[below left] {$W$};
|
|
|
|
\draw[color = purple, very thick, ->] (C) --+ (1,-3);
|
|
\draw[color = purple, very thick, ->] (7,1) --+ (-3,3);
|
|
\draw[color = purple] (4,4) node {$\bullet$} node[below left] {$X$};
|
|
\fi
|
|
\end{tikzpicture}
|
|
\end{center}
|
|
|
|
\begin{parts}
|
|
\part Le point $U$ tel que $\vec{AU} = \vec{BC}$.
|
|
\begin{solution}
|
|
En bleu
|
|
\end{solution}
|
|
\part Le point $V$ tel que $\vec{VA} = \vec{CD}$.
|
|
\begin{solution}
|
|
En rouge
|
|
\end{solution}
|
|
\part Le point $W$ tel que $\vec{BW} = -\vec{AC}$.
|
|
\begin{solution}
|
|
En vert
|
|
\end{solution}
|
|
\part Le point $X$ tel que $\vec{CZ} = \vec{BC} + \vec{CD}$.
|
|
\begin{solution}
|
|
En violet
|
|
\end{solution}
|
|
\part Est-il vrai que $2\vec{u} + \vec{BA} + \vec{DA} = \vec{0}$? \textit{La justification peut être un dessin ici.}
|
|
\begin{solution}
|
|
\TODO{à faire}
|
|
\end{solution}
|
|
\end{parts}
|
|
|
|
\pagebreak
|
|
\question[6]
|
|
\begin{parts}
|
|
\part Voici le tableau de variation de la fonction $f$
|
|
|
|
\begin{tikzpicture}
|
|
\tkzTabInit[espcl=2]{$x$/1,$f(x)$/1}{-4, -2, 0, 1, 2, 4}
|
|
\tkzTabVar{-/{-4}, +/{1}, -/{-3}, +/{0}, -/{-3}, +/{3}}
|
|
\end{tikzpicture}
|
|
|
|
\begin{subparts}
|
|
%1pt
|
|
\subpart Sur quels intervalles la fonction $f$ est-elle décroissante?
|
|
\begin{solution}
|
|
D'après le tableau de variations, la fonction $f$ est décroissante sur $\intFF{-2}{0} \cup \intFF{1}{2}$.
|
|
\end{solution}
|
|
%1pt
|
|
\subpart Déterminer le maximum de la fonction sur l'intervalle $\intFF{-4}{4}$.
|
|
\begin{solution}
|
|
Sur l'intervalle $_intFF{-4}{4}$ (l'intervalle de définition de $f$), le maximum est atteint pour $x = 4$ et vaut $f(4) = 3$.
|
|
\end{solution}
|
|
%1pt
|
|
\subpart Tracer une fonction qui a ce tableau de variation.
|
|
\begin{solution}
|
|
Voici une courbe possible
|
|
\begin{center}
|
|
\begin{tikzpicture}[scale=0.8]
|
|
\repere{-6}{6}{-6}{6}
|
|
\draw[very thick, color=red] plot [smooth,tension=0.5, mark=*] coordinates{%
|
|
(-4, -4) (-2,1) (0,-3) (1,0) (2,-3) (4,3)
|
|
};
|
|
\draw (4,3) node[above right] {$\mathcal{C}_f$};
|
|
\end{tikzpicture}
|
|
\end{center}
|
|
\end{solution}
|
|
\end{subparts}
|
|
|
|
\part Voici la représentation graphique de la fonction $g$.
|
|
|
|
\begin{center}
|
|
\begin{tikzpicture}[scale=0.8]
|
|
\repere{-6}{6}{-6}{6}
|
|
\draw[very thick, color=red] plot [smooth,tension=0.5, mark=*] coordinates{%
|
|
(-4, -4) (-3.5, 0) (-3, 4) (-2, 1) (-1, 0) (0, -1) (1, -2) (2, 1) (3, 3) (4, 2)%
|
|
};
|
|
\draw (4,3) node[above right] {$\mathcal{C}_g$};
|
|
|
|
\ifprintanswers
|
|
\draw[color = blue, dashed, very thick] (3,0) node[below] {3} -- (3,3) -- (0,3) node[left] {$f(3)$};
|
|
|
|
\draw[color=green, dashed, very thick] (-6,1) -- (6,1) node [above] {$y=1$};
|
|
\draw[color=green, dashed, very thick] (-3.4,1) node {$\bullet$} -- (-3.4,0) node [below] {-3,4};
|
|
\draw[color=green, dashed, very thick] (-2,1) node {$\bullet$} -- (-2,0) node [below] {-2};
|
|
\draw[color=green, dashed, very thick] (2,1) node {$\bullet$} -- (2,0) node [below] {2};
|
|
\fi
|
|
\end{tikzpicture}
|
|
\end{center}
|
|
|
|
\begin{subparts}
|
|
% O.5pt
|
|
\subpart Quel est l'image de 3 par cette fonction? Vous laisserez les traits de construction qui vous ont permis de répondre.
|
|
\begin{solution}
|
|
L'image de 3 par la fonction $g$ est 3. Voir les traits en bleu.
|
|
\end{solution}
|
|
% O.5pt
|
|
\subpart Quels sont les antécédents de 1 par cette fonction? Vous laisserez les traits de construction qui vous ont permis de répondre.
|
|
\begin{solution}
|
|
Les antécédents de 1 par cette fonction sont 2, -2 et environ -3,4. Voir les traits en vert.
|
|
\end{solution}
|
|
%2pts
|
|
\subpart Tracer le tableau variation de $f$.
|
|
\begin{solution}
|
|
Tableau de variation de cette fonction
|
|
|
|
\begin{tikzpicture}
|
|
\tkzTabInit[espcl=2]{$x$/1,$f(x)$/1}{-4, -3, 1, 3, 4}
|
|
\tkzTabVar{-/{-4}, +/{4}, -/{-2}, +/{3}, -/{2}, }
|
|
\end{tikzpicture}
|
|
|
|
\end{solution}
|
|
\end{subparts}
|
|
\end{parts}
|
|
|
|
\pagebreak
|
|
\question[6]
|
|
Cet exercice est une questionnaire à choix multiplies (QCM).
|
|
|
|
Dans le tableau ci dessous, \textbf{entourer} une réponse V (vrai) ou F (faux) à droite de chaque affirmation, en vous référant aux données situées dans la colonne de gauche.
|
|
|
|
\begin{itshape}
|
|
Une bonne réponse rapporte 0,5 points et une mauvaise réponse retire 0,25 point. L'absence de réponse ne donne ni n'enlève de point.
|
|
\end{itshape}
|
|
|
|
\hspace{-2cm}
|
|
\ifprintanswers
|
|
\includegraphics[scale=0.75]{./fig/QCM_coor}
|
|
\else
|
|
\includegraphics[scale=0.75]{./fig/QCM}
|
|
\fi
|
|
|
|
|
|
|
|
|
|
\end{questions}
|
|
|
|
|
|
\end{document}
|
|
|
|
%%% Local Variables:
|
|
%%% mode: latex
|
|
%%% TeX-master: "master"
|
|
%%% End:
|
|
|